LSAT and Law School Admissions Forum

Get expert LSAT preparation and law school admissions advice from PowerScore Test Preparation.

 Administrator
PowerScore Staff
  • PowerScore Staff
  • Posts: 8917
  • Joined: Feb 02, 2011
|
#80567
Complete Question Explanation

Flaw. The correct answer choice is (B).

Answer choice (A):

Answer choice (B): This is the correct answer choice.

Answer choice (C):

Answer choice (D):

Answer choice (E):

This explanation is still in progress. Please post any questions below!
User avatar
 paytenpar2014
  • Posts: 4
  • Joined: May 05, 2021
|
#86843
Hi, would someone please explain this one to me? I originally chose D because I was thinking that maybe it was a percent vs amount flaw.

Thanks!
User avatar
 Poonam Agrawal
PowerScore Staff
  • PowerScore Staff
  • Posts: 71
  • Joined: Apr 23, 2021
|
#86863
Hi Payten!

The key to understanding the flaw in this stimulus is to track the way Lindsey's argument flows.

Sentence 1: Several people claim that the company has been unfair in its bonus-giving practices.
Sentence 2: Maybe the people's claims are driven by the company's promise.
Sentence 3: The condition required for the promise was not met.
Sentence 4: The company did not need to keep the promise - so it acted fairly.

Notice that in the second sentence, Lindsey expresses uncertainty about the reason behind the people's claims of unfairness. In the third sentence, all she does is prove that one possible reason for people's claims is not valid. But in the fourth sentence, or the conclusion of this stimulus, Lindsey says with certainty that the company acted fairly - without addressing any of the other reasons that people could have for making their claims. Maybe there really was another reason for the unfairness claims - we just don't know.

Therefore, answer choice (B) is the correct answer because Lindsey only undermines one potential reason for the people's claims and uses that to invalidate their entire opinion.
 afulbright.2000@gmail.com
  • Posts: 13
  • Joined: Apr 28, 2021
|
#95605
I understand why B was the correct answer here, but can one explain why A is not correct?

Is it because the opinions of such individuals are not the point of the argument, but rather that their opinions were not further explained/supported before being negated by the author?

Thanks in advance!
 Rachael Wilkenfeld
PowerScore Staff
  • PowerScore Staff
  • Posts: 1358
  • Joined: Dec 15, 2011
|
#95618
Hiya afulbright,

Exactly. The author here doesn't agree with the opinions of the unnamed people that the company was unfair. In fact, Lindsey's conclusion is that the opinion of the unnamed people is incorrect. The argument doesn't rely on the opinion of unnamed people but instead is arguing AGAINST those very people. Lindsey doesn't need to do anything to show that the unnamed people were correct, because her conclusion is that they are incorrect in their opinion. So we can eliminate answer choice (A) because it is not accurately describing what happened in the stimulus.

Hope that helps!
 oliviaguerra7
  • Posts: 7
  • Joined: Aug 03, 2023
|
#102827
Hello!

I chose answer choice E instead of B. But after reading the review posts, I am now seeing that answer choice E is trying to strengthen Lindseys argument rather than point out a flaw. Is this reasoning accurate?

Thank you for the help!
 Luke Haqq
PowerScore Staff
  • PowerScore Staff
  • Posts: 742
  • Joined: Apr 26, 2012
|
#102888
Hi oliviaguerra7!

In this stimulus, the author indicates that if one thing happened (increased profits over last year), then the company would offer raises. The first thing didn't happen, so the author concludes that it's fair that there were no raises. But what if there were other reasons that raises were merited? Perhaps the staff worked extra hard, or perhaps the profits were far more than they expected. It's possible that the company acted unfairly for a variety of reasons rather than only the one mentioned.

I wouldn't say that answer choice (E) strengthens the argument. Rather, it can be eliminated because it doesn't accurately describe a flaw. I could be missing something, though--feel free to unpack your rationale for how you see it strengthening the stimulus and we can address that reasoning more in depth!
User avatar
 npant120
  • Posts: 20
  • Joined: Aug 27, 2023
|
#105079
I was reviewing the different types of flawed argumentation earlier and was just wondering - is this an example of flawed use of evidence? Specifically that Lindsey uses some evidence for a claim (profits were smaller this year than last year) to conclude that the claim is definitely true (company acted fairly)?
User avatar
 Jeff Wren
PowerScore Staff
  • PowerScore Staff
  • Posts: 389
  • Joined: Oct 19, 2022
|
#105164
Hi npant120,

You've generally got the right idea, but technically Lindsey uses some evidence against a claim (That claim being the company was unfair when it didn't give bonuses) to conclude that the claim is false (i.e. the company didn't act unfairly, meaning that the company did act fairly).

While it is equivalent to your explanation, the actual description in Answer B is referring to the claim that the argument attacks, which is why the answer uses the words "false" and "undermined."

Get the most out of your LSAT Prep Plus subscription.

Analyze and track your performance with our Testing and Analytics Package.